LSAT and Law School Admissions Forum

Get expert LSAT preparation and law school admissions advice from PowerScore Test Preparation.

 Administrator
PowerScore Staff
  • PowerScore Staff
  • Posts: 8919
  • Joined: Feb 02, 2011
|
#23219
Complete Question Explanation

Parallel Reasoning-SN. The correct answer choice is (B)

This stimulus is easier to understand if you pay attention to the formal logic content:

Premises:
  • Rhonda goes to movie ..... :arrow: ..... Paul goes to concert

    Paul goes to concert ..... :arrow: ..... Ted goes to concert

    Ted refuses to go to the concert
Conclusion:
  • Ted goes to the concert ..... :arrow: ..... Rhonda goes to the movie
The reasoning is logically valid, and involves the contrapositive of the additive inference.

Answer choice (A): This answer choice does not link the first element to the last, so is dissimilar. Furthermore, this choice involves a Mistaken Reversal of the premise:
  • Janice visits ..... :arrow: ..... Janice finds baby-sitter
so this choice is logically flawed, and incorrect.

Answer choice (B): This is the correct answer choice. It can be diagrammed as:
  • Gary does laundry ..... :arrow: ..... Peter has to work ..... :arrow: ..... Cathy is ill

    Cathy is ill ..... :arrow: ..... Gary does laundry
This illustrates that this response involves a correct additive inverse and uses the contrapositive, just as was done in the stimulus.

Answer choice (C): Since the first premise contains two sufficient conditions, this response fails the Premise Test, and is incorrect. You should immediately eliminate this choice.

Answer choice (D): Since the first premise contains a choice of necessary elements, and the stimulus did not, this response fails the Premise Test. Eliminate this choice immediately.

Answer choice (E): This response does not link the first and last elements. Also, this answer choice contains a Mistaken Reversal of
  • Mark goes to the museum ..... :arrow: ..... Postponed appointments
so this choice contains fallacious reasoning, and is incorrect.
 avengingangel
  • Posts: 275
  • Joined: Jun 14, 2016
|
#28655
After correctly identifying the structure of the reasoning in the stimulus, I immediately went thru and first eliminated any answer choice that didn't have 'unless' included. So after I quickly eliminated C D & E, I had just A & B left as my Contenders. I felt the use of 'unless' was important (enough) in matching up the answer choices... is that incorrect?? Is it not necessary as a part to compare when paralleling reasoning ??? Thanks.
 Nikki Siclunov
PowerScore Staff
  • PowerScore Staff
  • Posts: 1362
  • Joined: Aug 02, 2011
|
#28862
Angel,

I'm afraid this is not a logically correct way to eliminate the incorrect answer choices here. You just lucked out :) The reason is simple: you should be looking for an answer choice that has the same conditional structure as the original argument, and the word "unless" can easily convey a conditional relationship of the type you're looking for. There are many ways of conveying the same exact set of conditional relationships, so don't just dismiss answers that use conditional indicators that differ from the ones used in the stimulus. This is not a material difference.

For more information on this issue, please consult the blog post I wrote not too long ago :)

Thanks!
 avengingangel
  • Posts: 275
  • Joined: Jun 14, 2016
|
#29578
Nikki, thanks again. I just re-visited this question, and this time I:

1) Read thru the stimulus and quickly recognized the logic structure as A :arrow: B :arrow: C; conclusion: ~C :arrow: ~A (I didn't even have to write it out, like I usually would!).
2) I then quickly glanced at the conclusion of each answer choice to check if it matches with the negation of the stimulus' conclusion, leaving me with B & C as contenders
3) Then I read thru B, decided it was correct bc it matched up perfectly with my prephrase.
4) Read thru the entirety of C, saw that it was wrong, even from the first sentence.
5) Chose B as the correct answer.

It didn't even cross my mind that 'unless' had to be in the answer choice, just that the diagram had to be the same! That's encouraging that from the time I posted this (a month ago), I've since addressed that parallel reasoning question mis-approach.

Just want to make sure, though, that my first step in eliminating the answer choices with a conclusion that does not "match" the negation(/certainty) of the stimulus' conclusion is a good & accurate way to approach it?? I did that since the Course book recommends that as one of the first things to look for to help you quickly assign answer choices as losers or contenders. I do, however, feel like I remember reading that, if after you do that, your contenders don't seem to be correct, you should re-assess the answer choices you eliminated, bc there is still a (small) possibility it could be the right answer... so, if B or C didn't seem right, I would have looked back at A, D, & E. But since I was sure B was right, I felt good about that approach in this question. Look forward to your thoughts. Thanks !!
 Claire Horan
PowerScore Staff
  • PowerScore Staff
  • Posts: 408
  • Joined: Apr 18, 2016
|
#29601
Avengingangel,

You restated the strategy correctly. You should eliminate answer choices that don't seem to match, but there is a small chance that you made a mistake in thinking a conclusion didn't match that did. It isn't so much that you should eliminate answer choices that don't have exactly the same wording, but rather the level of certainty, etc. needs to be the same. Good job!
 mankariousc
  • Posts: 32
  • Joined: Feb 13, 2017
|
#34677
Hello!

I got this question right, but I was confused about one of the wrong answer choices. On C, the explanation says that there are two necessary conditions. I thought there were two sufficient conditions because both "it doesn't rain" and the "market has fresh fist trout" are introduced by "if" which I thought was a sufficient indicator. Could you help me with this?
 Jon Denning
PowerScore Staff
  • PowerScore Staff
  • Posts: 904
  • Joined: Apr 11, 2011
|
#34753
Hi mankariousc - thanks for the question!

You are correct about answer choice C: that should say two sufficient conditions, not two necessary conditions. So I'm going to edit the explanation provided to make that change :)

Good eye and thanks for letting us know! Keep up the hard work!
 Bruin96
  • Posts: 33
  • Joined: Sep 04, 2019
|
#68197
I was able to do this question correctly. However, I wanted to know if it would be correct to eliminate answer choices that contained different language from that of the stimulus. When first reviewing the answers I eliminated A, D, E as their conclusions all stated that they "will" and the stimulus stated "will not"
 Adam Tyson
PowerScore Staff
  • PowerScore Staff
  • Posts: 5153
  • Joined: Apr 14, 2011
|
#68598
Great question, Bruin96! The important aspect to parallel in the conclusions is not the specific language, but the strength and type of that language. While "will" and "will not" are logical opposites, they are the same type and strength of language - factual and certain (as opposed to opinion-based or about either possibility of probability). Thus, it IS possible to parallel a "will" stimulus with a "will not" answer choice, so long as the rest of the structure of the argument is the same (in this case, a contrapositve of a conditional chain of three terms). Most of the time, the "polarity" of the correct answer does match that of the stimulus (positive terms matched with positive, negatives with negatives), but it doesn't have to.

Add that to your toolbelt, and keep at it!

Get the most out of your LSAT Prep Plus subscription.

Analyze and track your performance with our Testing and Analytics Package.